If V is more popular than Q and J is less popular than L, then which one of the following could be true of the ranking?

Theresaturner on August 15, 2015

Please help

Can you please explain the answer to this question

Replies
Create a free account to read and take part in forum discussions.

Already have an account? log in

Naz on September 2, 2015

You can access the video explanation by tapping the "play" button on the screen.

Hope that clears things up! Please let us know if you have any other questions.

jaish1021 on March 23, 2018

The print out work books that were sent to me have a different question. Are they not consistent?

muralikrishnag2004 on May 31, 2019

could you please explain me the 10 question of sequencing games

Ravi on May 31, 2019

Hey there,

Feel free to direct any support related issues to our
support staff by tapping "support" from the left menu or by calling
855.483.7862 ext. 2 Monday-Friday 9am-6pm PT.

Please save the boards for content-related questions. Thanks!

Happy to help explain this question.

The question asks, "If V is more popular than Q and J is less popular
than L, then which one of the following could be true of the ranking?"

Based on the rules, we know we have a 7-slot game.

_ _ _ _ _ _ _

We also know that J and L aren't as popular as H

H-J and H-L

J is more popular than Q

J-Q

S and V are each less popular than L

L-S and L-V

P and S are each less popular than Q

Q-S and Q-P

S isn't 7th

Combining these rules with the additional info we've been given in the
question, we know that if J isn't as popular as L, then L has to be
ranked second. This is because J and L are both leaders of all of the
other pieces in the game, and H has to go first since it's before both
of them.

If V is more popular than Q, then this means that both V and J are
after L but before Q. This means that V and J could be in either 3 or
4. Q has to be 5th. Because S can't go in 7, then S has to be 6th.
This leaves P for the 7th spot. Because J and V could each go in 3 or
4, then this means that (D) could be true, as it states that J is more
popular than V. Thus, (D) is the correct answer choice.

Does this make sense? Let us know if you have any more questions!